Compute Tricky Limit Using Taylor Series and De L'hopital's Theorem

In summary, a tricky limit is a complex mathematical problem that involves finding the value of a limit in an unconventional way. This often requires the use of advanced techniques such as Taylor series. A Taylor series is an infinite sum of terms that can be used to represent a function or approximate it with a polynomial. To apply Taylor series to solve a tricky limit, one must find the derivatives of the function at the given point and plug them into the Taylor series formula. Common mistakes when using Taylor series include not considering the remainder term and using the wrong number of terms in the series. However, Taylor series can only be used to solve certain types of tricky limits, and in some cases, other techniques may be more effective.
  • #1
Felafel
171
0

Homework Statement



compute the following limit:

## \displaystyle{\lim_{x\to +\infty} x \left((1+\frac{1}{x})^{x} - e \right)} ##

The Attempt at a Solution



i wanted to use the taylor expansion, but didn't know what ##x_0## would be correct, as the x goes to ## \infty##.

also, i tried to use de l'hopital's theorem but it wouldn't work.
how can i do that?
 
Physics news on Phys.org
  • #2
Is it supposed to be e or e^x?
 
  • #3
Surely e; this limit would quantify, to first order, how rapidly [itex](1 + 1/x)^x[/itex] converges to its limit e.
 
  • #4
Try rewriting the limit in terms of z=1/x.
 
  • #5
[tex]x\left(\left(1 + \frac1x\right)^x - e\right) = x\left(\exp\left(x\ln \left(1 + \frac1x\right)\right) - e\right)[/tex]

At this point, one can proceed to substitute the Maclaurin series for [itex]\ln(1 + t)[/itex] with [itex]t = x^{-1}[/itex] (we want the limit as [itex]x \to \infty[/itex], so we can assume [itex]0 < t = x^{-1} < 1[/itex]) to get
[tex]x\ln\left(1 + \frac1x\right) = \sum_{n=0}^{\infty} \frac{(-1)^n}{(n+1)x^n}[/tex]
and then substitute that into the series for [itex]\exp(x)[/itex] to get
[tex]\exp\left(x\ln\left(1 + \frac1x\right)\right) =
\sum_{k=0}^{\infty} \frac1{k!} \left(\sum_{n=0}^{\infty} \frac{(-1)^n}{(n+1)x^n}\right)^k = \sum_{m=0}^{\infty} \frac{a_m}{x^m}[/tex]
(everything here is absolutely convergent, so I can add terms in whatever order I want) so that
[tex]x\left(\left(1 + \frac1x\right)^x - e\right) = \left(\sum_{m=0}^{\infty} \frac{a_m}{x^{m-1}} - ex\right) = x(a_0 - e) + a_1 + \sum_{m=2}^{\infty} \frac{a_m}{x^{m-1}}[/tex]
It's clear that the series on the right tends to 0 as [itex]x \to \infty[/itex], so all you need to work out to determine the limit (if any) is [itex]a_0[/itex] and [itex]a_1[/itex]. (You'll need to consider, for each [itex]k[/itex], what the coefficients of [itex]x^0[/itex] and [itex]x^{-1}[/itex] are and add them all together. Actually you can truncate the inner series after [itex]n = 1[/itex], because including [itex]n \geq 2[/itex] doesn't give you any more terms of order [itex]x^0[/itex] and [itex]x^{-1}[/itex] then you already have.

But that is a brute-force method and I'm sure there's a more elegant solution.
 
  • #6
Felafel said:

Homework Statement



compute the following limit:

## \displaystyle{\lim_{x\to +\infty} x \left((1+\frac{1}{x})^{x} - e \right)} ##

The Attempt at a Solution



i wanted to use the taylor expansion, but didn't know what ##x_0## would be correct, as the x goes to ## \infty##.

also, i tried to use de l'hopital's theorem but it wouldn't work.
how can i do that?
L'Hôpital's rule works just fine.

[itex]\displaystyle \frac{d}{dx}\left(f(x)\right)^x=\frac{d}{dx}e^{x \ln(f(x))}[/itex]
[itex]\displaystyle =\left(\ln(f(x))+\frac{x}{f(x)}\right)e^{x \ln(f(x))}[/itex]

[itex]\displaystyle =\left(\ln(f(x))+\frac{xf'(x)}{f(x)} \right)\left(f(x)\right)^x[/itex]

Fixed in Edit.
 
Last edited:
  • #7
SammyS said:
L'Hôpital's rule works just fine.

[itex]\displaystyle \frac{d}{dx}\left(f(x)\right)^x=\frac{d}{dx}e^{x \ln(f(x))}[/itex]
[itex]\displaystyle =\left(\ln(f(x))+\frac{x}{f(x)}\right)e^{x \ln(f(x))}[/itex]

[itex]\displaystyle =\left(\ln(f(x))+\frac{x}{f(x)} \right)\left(f(x)\right)^x[/itex]
You forgot to apply the chain rule when differentiating ln(f(x)).
 
  • #8
vela said:
you forgot to apply the chain rule when differentiating ln(f(x)).
DUH !

Thank you vela !
 
  • #9
what if i use the taylor series for ##x_0=1##?
it should become:
##\displaystyle \lim_{x \to \infty}\ x [(2+(x-1)log(4)-1+o((x-1)^2)-e] = \infty * \infty = \infty ##

is it correct?
 
  • #10
Nope
 
  • #11
..maybe it was just de l'hospital then, does this look ok to you?

##\displaystyle \lim{x \to \infty}\ \frac{1}{\frac{1}{x}} ((1+ \frac{1}{x})^x-e) = \frac{0}{0}##
using de l'hopital
##\displaystyle \lim{x \to \infty}\ \frac{1}{\frac{-1}{x^2}} (x(1+\frac{1}{x})^{x-1}*\frac{-1}{x^2}##
which is
## = \infty * \frac{e}{1+0} = \infty ##
 
  • #12
You didn't differentiate correctly. See post #6. You should get a finite answer.
 
Last edited:
  • #13
i can't solve it with derivatives. i eep using de l'hopital but it gets worse and worse.
i thought i could split it into:
##\displaystyle \lim x \to \infty\ x(1+1/x)^x - \displaystyle \lim x \to \infty\ xe ##
and use taylor series for x=0 of the second part, which becomes:
##x*\displaystyle\sum\limits_{k=0}^n \frac{1}{k!} ##
however, i can't figure out the series of the first part
 
  • #14
You don't want to use a Taylor series about x=0. You want to be able to neglect high-order terms which you can't because the limit is for ##x \to \infty##.

Why don't you try my suggestion of rewriting the limit in terms of z=1/x? It'll make applying L'Hopital's more straightforward, and because the limit will be for ##z \to 0##, you can use a Taylor series about ##z=0## if necessary.
 
  • #15
okay, I think I've solved it:
##\lim_{y \to 0}\frac{(1+y)^{\frac{1}{y}}-e}{y}=\lim_{y \to 0} \frac{e^{\frac{1}{y}log(1+y)}-e}{y}##
I find the Maclaurin extension for log(1+y), stopping at the third power

##log(1+y)=y-\frac{y^2}{2}+\frac{y^3}{3}##

##\displaystyle{\lim_{y \to 0} \frac{e^{1-\frac{y}{2}-\frac{y^2}{3}}-e }{y}}(=\frac{0}{0})##

## \stackrel{\text{H}}{=} \displaystyle{ \lim_{y \to 0} (-\frac{1}{2}+2\frac{y}{3})e^{1-\frac{y}{2}+\frac{y^2}{2}}}##

##=-\frac{e}{2}##

should be ok now
 
  • #16
Yes, that's ok now. There's a couple of what are obviously just typos. But nothing important.
 

1. What is a tricky limit?

A tricky limit is a type of mathematical problem that involves finding the value of a limit in a complex or unconventional way. It often requires the use of advanced mathematical techniques, such as Taylor series, to solve.

2. What is a Taylor series?

A Taylor series is a way to represent a function as an infinite sum of terms. It is often used in calculus to find the value of a function at a particular point, or to approximate a function with a polynomial.

3. How do I apply Taylor series to solve a tricky limit?

To apply Taylor series, you first need to find the derivatives of the function at the given point. Then, you can plug these derivatives into the Taylor series formula and simplify to find the value of the limit.

4. What are some common mistakes when using Taylor series to solve tricky limits?

One common mistake is not considering the remainder term in the Taylor series, which can lead to an inaccurate approximation of the limit. Another mistake is using the wrong number of terms in the series, which can also result in an incorrect answer.

5. Can Taylor series be used to solve all tricky limits?

No, Taylor series can only be used to solve certain types of tricky limits. In some cases, other techniques such as L'Hopital's rule may be more effective in finding the value of the limit.

Similar threads

  • Calculus and Beyond Homework Help
Replies
17
Views
588
  • Calculus and Beyond Homework Help
Replies
2
Views
1K
  • Calculus and Beyond Homework Help
Replies
2
Views
699
  • Calculus and Beyond Homework Help
Replies
1
Views
821
  • Calculus and Beyond Homework Help
Replies
27
Views
2K
  • Calculus and Beyond Homework Help
Replies
1
Views
224
  • Calculus and Beyond Homework Help
Replies
2
Views
1K
  • Calculus and Beyond Homework Help
Replies
2
Views
165
  • Calculus and Beyond Homework Help
Replies
2
Views
75
  • Calculus and Beyond Homework Help
Replies
6
Views
2K
Back
Top